Why is arcsin converted into logs in this equation?

  • Thread starter Thread starter cathy
  • Start date Start date
  • Tags Tags
    Turning
Click For Summary
The equation arcsinh(e^x) = ln(e^x + √(e^(2x) + 1) demonstrates the relationship between the inverse hyperbolic sine function and natural logarithms. This conversion is based on the definition of the arcsinh function, which can be expressed in logarithmic terms. The discussion includes a derivation showing how to manipulate the expression to reveal the logarithmic form. Participants clarify the steps involved in solving for y in terms of x, linking the hyperbolic sine function to its logarithmic representation. Understanding this relationship is essential for solving related mathematical problems effectively.
cathy
Messages
90
Reaction score
0

Homework Statement



Hello. Will someone please explain to me why this is true?
arcsinh(e^x) = ln(e^x + √(e^(2x) + 1))
2. The attempt at a solution

I cannot figure out why arcsin is able to be put into terms of ln. Thank you in advance.
 
Physics news on Phys.org
hi caty! :smile:

(hey, what's an h ? :wink:)

2sinh[ln(ex + √(e2x + 1))]

= exp[ln(ex + √(e2x + 1))] - exp[-ln(ex - √(e2x + 1))]

= ex + √(e2x + 1)) - 1/[ex + √(e2x + 1))]

= [e2x + e2x + 1 + 2ex√(e2x + 1)) - 1]/[ex + √(e2x + 1))]

= 2ex :wink:

alternatively, put ex = sinhy

then sinh[ln(ex + √(e2x + 1))]

= sinh[ln(sinhy + coshy)]

= sinh[ln(ey)]

= sinh[y] = ex
 
  • Like
Likes 1 person
In addition to Tiny's explanation you could also note ##y =\sinh^{-1}(e^x)## is the same as ##e^x =\sinh(y)=
\frac{e^y - e^{-y}} 2## or ##2e^x = e^y - \frac 1 {e^y}##. Solve that for ##y## in terms of ##x## and you will get your expression and you will see where the logarithms come from.
 
Last edited:
  • Like
Likes 1 person
Ahh got it! Thank you very much!
 
Question: A clock's minute hand has length 4 and its hour hand has length 3. What is the distance between the tips at the moment when it is increasing most rapidly?(Putnam Exam Question) Answer: Making assumption that both the hands moves at constant angular velocities, the answer is ## \sqrt{7} .## But don't you think this assumption is somewhat doubtful and wrong?

Similar threads

  • · Replies 5 ·
Replies
5
Views
1K
Replies
2
Views
2K
  • · Replies 18 ·
Replies
18
Views
4K
  • · Replies 20 ·
Replies
20
Views
3K
  • · Replies 1 ·
Replies
1
Views
1K
Replies
18
Views
4K
  • · Replies 5 ·
Replies
5
Views
2K
  • · Replies 4 ·
Replies
4
Views
1K
  • · Replies 7 ·
Replies
7
Views
4K
  • · Replies 10 ·
Replies
10
Views
2K